subject
Physics, 30.07.2020 09:01 issacurlyheadka

Help me please, it's so hard


Help me please, it's so hard
Help me please, it's so hard

ansver
Answers: 1

Another question on Physics

question
Physics, 22.06.2019 00:30
Part f - example: finding two forces (part i) two dimensional dynamics often involves solving for two unknown quantities in two separate equations describing the total force. the block in (figure 1) has a mass m=10kg and is being pulled by a force f on a table with coefficient of static friction îľs=0.3. four forces act on it: the applied force f (directed î¸=30â above the horizontal). the force of gravity fg=mg (directly down, where g=9.8m/s2). the normal force n (directly up). the force of static friction fs (directly left, opposing any potential motion). if we want to find the size of the force necessary to just barely overcome static friction (in which case fs=îľsn), we use the condition that the sum of the forces in both directions must be 0. using some basic trigonometry, we can write this condition out for the forces in both the horizontal and vertical directions, respectively, as: fcosî¸â’îľsn=0 fsinî¸+nâ’mg=0 in order to find the magnitude of force f, we have to solve a system of two equations with both f and the normal force n unknown. use the methods we have learned to find an expression for f in terms of m, g, î¸, and îľs (no n).
Answers: 2
question
Physics, 22.06.2019 02:20
Which changes of state do the labels represent? a: b: c:
Answers: 1
question
Physics, 22.06.2019 05:10
Which situation will have the highest resistance? a.long wire and high temperatureb.short wire and high temperaturec.long wire and cold temperaturedshort wire and low temperature
Answers: 2
question
Physics, 22.06.2019 12:10
Point charges q1 = 51 µc and q2 = −27 µc are placed 1.0 m apart, with q2 on the right. what is the force (in n) on a third charge q3 = 19 µc placed midway between q1 and q2? (assume the positive direction is to the right. indicate the direction with the sign of your answer.)
Answers: 3
You know the right answer?
Help me please, it's so hard

...
Questions
Questions on the website: 13722363